The midpoint of a segment is (6,−4) and one endpoint is (13,−2). Find the coordinates of the other endpoint.

A. (20, -6)
B. (-1, 0)
C. (-1, -6)
D. (20, 0)

Answers

Answer 1

Answer:

(-1,-6)

Step-by-step explanation:

(13 + x)/2 = 6

13+x= 12

x = -1

~~~~~~~~~~~~~~~

(-2 + y ) / 2 = -4

-2 + y = -8

y = -6

Answer 2

The coordinates of the other endpoint will be (-1,-6). The correct option is C.

What is the midpoint of the line?

Divide the measurement of the distance between the two end locations by 2. The middle of that line is located at this separation from either end.

A coordinate plane is a 2D plane that is formed by the intersection of two perpendicular lines known as the x-axis and y-axis.

Given that the midpoint of a segment is (6,−4) and one endpoint is (13,−2).

The x- coordinate will be calculated as:-

(13 + x)/2 = 6

13+x= 12

x = -1

The y-coordinate will be calculated as:-

(-2 + y ) / 2 = -4

-2 + y = -8

y = -6

Therefore, the coordinates of the other endpoint will be (-1,-6). The correct option is C.

To know more about midpoints of the line follow

https://brainly.com/question/24431553

#SPJ2


Related Questions

In figure above, if l1 | | l2 then value of x is:
a) 40°
b) 50°
c) 80°
d) 100°

Answers

Answer:

its letter c so 80

Step-by-step explanation:

I hope this help

solve for x.
solve for x.
solve for x.

Answers

Answer:

[tex]x=10[/tex]

Step-by-step explanation:

A secant is a line segment that intersects a circle in two places. One property of a secant is the product of the lengths ratio. This ratio can be described as the following, let ([tex]inside[/tex]) refer to the part of the secant that is inside the circle, and ([tex]outside[/tex]) refer to the part that is outside of it. ([tex]total[/tex]) will refer to the entirety of the secant or ([tex]inside+outside[/tex]). The numbers (1) and (2) will be used as subscripts to indicate that there are two different secants.

[tex](outside_1)(total_2)=(outside_2)(total_2)[/tex]

Substitute,

[tex](outside_1)(total_2)=(outside_2)(total_2)[/tex]

[tex](outside_1)(inside_1+outisde_1)=(outside_2)(inside_2+outside_2)[/tex]

[tex](6)(6+x+5)=(7)(7+x+1)[/tex]

Simplify,

[tex](6)(6+x+5)=(7)(7+x+1)[/tex]

[tex]6(x+11)=7(x+8)[/tex]

[tex]6x+66=7x+56[/tex]

Inverse operations,

[tex]6x+66=7x+56[/tex]

[tex]66=x+56[/tex]

[tex]10=x[/tex]

after allowing 20% discount an article is sold for rs.672 levying 12% VAT, find its market price​

Answers

The market price is Rs. 750 which was obtained by creating a mathematical relationship from the given parameters.

PERCENTAGE DISCOUNT = 20%

VAT LEVIED= 12%

PRICE SOLD = 672

Let the MARKET PRICE = m

Hence,

market price * (1 - discount) * (1 + VAT) = price sold

m * (1 - 20%) * (1 + 12%) = 672

m * (1 - 0.2) * (1 + 0.12) = 672

m * 0.8 * 1.12 = 672

0.896m = 672

m = 672 / 0.896

m = Rs. 750

Learn more :

https://brainly.com/question/20418815

The Market Price of the product is RS. 750.

The Market Price is calculated by dividing the components associated to Discount, which is less than 1, and the Value Added Tax, which more than 1, to the Resulting Price.

[tex]c_{M} = \frac{c_{R}}{\left(1-\frac{r_{D}}{100} \right)\cdot \left(1+\frac{r_{T}}{100} \right)}[/tex] (1)

Where:

[tex]c_{M}[/tex] - Market price, in monetary units.

[tex]c_{R}[/tex] - Resulting price, in monetary units.

[tex]r_{D}[/tex] - Discount rate, in percentage.

[tex]r_{T}[/tex] - Tax rate, in percentage.

If we know that [tex]c_{R} = 672[/tex], [tex]r_{D} = 20[/tex] and [tex]r_{T} = 12[/tex], then the market price is:

[tex]c_{M} = \frac{672}{\left(1-\frac{20}{100} \right)\cdot \left(1+\frac{12}{100} \right)}[/tex]

[tex]c_{M} = 750[/tex]

The market price of the product is RS. 750.

Simplify. (x2+2x-4)+(2x-5x-3)​

Answers

Answer:

Step by Step Solution

More Icon

STEP

1

:

3

Simplify ——

x2

Equation at the end of step

1

:

3

((((2•(x2))-5x)-——)+2x)-3

x2

STEP

2

:

Equation at the end of step

2

:

3

(((2x2 - 5x) - ——) + 2x) - 3

x2

STEP

3

:

Rewriting the whole as an Equivalent Fraction

3.1 Subtracting a fraction from a whole

Rewrite the whole as a fraction using x2 as the denominator :

2x2 - 5x (2x2 - 5x) • x2

2x2 - 5x = ———————— = ———————————————

1 x2

Equivalent fraction : The fraction thus generated looks different but has the same value as the whole

Common denominator : The equivalent fraction and the other fraction involved in the calculation share the same denominator

STEP

4

:

Pulling out like terms

4.1 Pull out like factors :

2x2 - 5x = x • (2x - 5)

Adding fractions that have a common denominator :

4.2 Adding up the two equivalent fractions

Add the two equivalent fractions which now have a common denominator

Combine the numerators together, put the sum or difference over the common denominator then reduce to lowest terms if possible:

x • (2x-5) • x2 - (3) 2x4 - 5x3 - 3

————————————————————— = —————————————

x2 x2

Equation at the end of step

4

:

(2x4 - 5x3 - 3)

(——————————————— + 2x) - 3

x2

STEP

5

:

Rewriting the whole as an Equivalent Fraction :

5.1 Adding a whole to a fraction

Rewrite the whole as a fraction using x2 as the denominator :

2x 2x • x2

2x = —— = ———————

1 x2

Polynomial Roots Calculator :

5.2 Find roots (zeroes) of : F(x) = 2x4 - 5x3 - 3

Polynomial Roots Calculator is a set of methods aimed at finding values of x for which F(x)=0

Rational Roots Test is one of the above mentioned tools. It would only find Rational Roots that is numbers x which can be expressed as the quotient of two integers

Triangle XYZ is isosceles. The measure of the vertex angle, Y, is twice the measure of a base angle. What is true about triangle XYZ? Select three options.

Answers

Answer:

A. Angle Y is a right angle.

B. The measure of angle Z is 45°.

E. The perpendicular bisector of creates two smaller isosceles triangles.

Step-by-step explanation:

Let x represent the measures of base angles X and Z. 2x is the measure of vertex angle Y.

x + x + 2x = 180°

x = 45°

2x = m∠Y = 90°

The triangle is an isosceles right triangle which has base angles of 45°.

The perpendicular bisector of line XZ creates two smaller isosceles triangles with acute angles of 45°

Answer:

The answers are A B E

Step-by-step explanation:

(06.01)

Write the following expression in exponential form:

1.6 × 1.6 × 1.6 × 1.6

41.6
1.64
1.6 × 4
1.6 + 4

Answers

Answer:

[tex]1.6^{4}[/tex]

Step-by-step explanation:

1.6 is multiplied by itself 4 times. This is represented in exponential form as

[tex]1.6^{4}[/tex]

The graph of a line is shown below. What is the equation of the line, in slope-intercept form, that is parallel to this line and has a y-intercept of 1?

Answers

Answer:

[tex]y = - \frac{3}{2} x + 1[/tex]

Step-by-step explanation:

Slope -intercept form: y= mx +c, where m is the slope and c is the y-intercept.

Parallel lines have the same slope. Let's find the slope of the given line.

Given points: (-2, 0) and (0, -3)

[tex]\boxed{slope = \frac{y1 - y2}{x1 - x2} }[/tex]

slope of given line

[tex] = \frac{0 - ( - 3)}{ - 2 - 0} [/tex]

[tex] = \frac{0 + 3}{ - 2} [/tex]

[tex] = - \frac{3}{2} [/tex]

[tex]y = - \frac{3}{2} x + c[/tex]

Given that the y- intercept is 1, c= 1.

[tex]y = - \frac{3}{2} x + 1[/tex]

what is the answer? I need help!! please and thank you

Answers

Answer:

B

Step-by-step explanation:

27%=0.27 and sqrt(2)<2.75

help! please!!!!!! look at photo :))

Answers

Hey there!

We know that Danielle earns $10 per hour, so muliply that by 3 and get 30.

Because Danielle works an extra half an hour, divide 10 by 2 and get 5.

Danielle earns $35 in 3 hours and a half.

Hope this helps! Please mark me as brainliest!

Have a wonderful day :)


Charlene is a salesperson. Let y represent her total pay (in dollars). Let x represent the number of
items she sells. Suppose that x and y are related by the equation y=32x + 1900.
What is Charlene's total pay if she doesn't sell any items?
A. $32
B. $1,900
C. $3,200
D. $19

Answers

1,900
Multiply 32 by 0, and you get y=1,900

y= -(x+3)^2 -5
What is the leading coefficient?
How do you find the vertex?

Answers

Answer:

To find the leading coefficient, first expand the function:

[tex]y= -(x+3)^{2} -5\\\\y=-(x^{2} +6x+9)-5\\\\y=-x^{2} -6x-9-5\\\\y=-x^{2} -6x-14[/tex]

The leading coefficient is the coefficient of the highest-order term, which, in this case, would be the -1 from -x².

To find the vertex: see image below

Vertex = (-3, -5)

3. If triangle ABC has the following measurements, find the measure of angle A.
a = 17
b = 21
C = 25

Answers

9514 1404 393

Answer:

  (a)  42.3°

Step-by-step explanation:

Side 'a' is the shortest of three unequal sides, so angle A will be the smallest angle in the triangle. Its measure can be found from the Law of Cosines.

  a² = b² +c² -2bc·cos(A)

  cos(A) = (b² +c² -a²)/(2bc) = (21² +25² -17²)/(2·21·25) = 777/1050

  A = arccos(777/1050) ≈ 42.3°

The measure of angle A is about 42.3°.

_____

Additional comment

The smallest angle in a triangle can never be greater than 60°. This lets you eliminate choices that exceed that value.

Answer:

 (a)  42.3°

Step-by-step explanation:

Which graph represents the function f(x)=|x−1|−3 ?

Answers

Answer is B, (the one on the top right)

Jo bought a used car for $6000 and paid a 15% deposit. How much did he still have to pay?

Answers

Answer:

900 is the correct awnser

If YWZ=17, what is WXY?

34
56
17
73

Answers

Answer:

73

Step-by-step explanation:

17×2=34

180-34=146

146/2=73

=73

What is the difference between-5and2

Answers

Answer:

7

Step-by-step explanation:

Consider the absolute value of the difference , that is

| - 5 - 2 | = | - 7 | = 7

or

| 2 - (- 5) | = | 2 + 5 | = | 7 | = 7

Answer:

7

Step-by-step explanation:

Difference is - sign so the equation is: 2- -5 which is 7. Or

think a number line, -5 is 5 spots to 0, then two more spots to 2 so 5+2=7

Find m/ELM if m/ELM = 15x - 1, m/KLE = 20°, and m/KLM = 17x - 1.​

Answers

Answer:

∠ ELM = 149°

Step-by-step explanation:

∠ KLM = ∠ KLE + ∠ ELM  , substitute values

17x - 1 = 20 + 15x - 1

17x - 1 = 15x + 19 ( subtract 15x from both sides )

2x - 1 = 19 ( add 1 to both sides  )

2x = 20 ( divide both sides by 2 )

x = 10

Then

∠ ELM = 15x - 1 = 15(10) - 1 = 150 - 1 = 149°


What is the surface area of the right prism?
92 ft2
46 ft2
48 ft2
70 ft2

(will mark brainliest <3)

Answers

Answer:  70 ft^2   ( choice D )

=========================================================

Work Shown:

L = 8 ft = lengthW = 3 ft = widthH = 1 ft = height

SA = surface area of the rectangular prism (aka block or box)

SA = 2*(LW + LH + WH)

SA = 2*(8*3 + 8*1 + 3*1)

SA = 2*(24 + 8 + 3)

SA = 2*(35)

SA = 70 square feet

This is the amount of wrapping paper you would need to cover all six sides of the box. This assumes that there are no gaps or overlaps.

which statement is true

Answers

3) an $8 delivery fee and $1.50 per litre of water

This is because the $8 is a constant baseline, then adding $1.50 times the amount of litres purchased.

Select the two values of x that are roots of this equatio 2x - 5 = - 3x ^ 2

Answers

alright I can help!

so to find the two values of x that are roots of the equation we need to put the variables all on one side so that we can set up the quadratic formula.

3x^2+2x-5=0 (the -3x^2 becomes positive when moved across the equal sign)

now we can set up the quadratic formula. the equation is x= (-b+-(sqrt of b^2 -4ac))/ 2a

so now we just plug in our variables.

x= (-2+-(sqrt of 2^2 -4×3×-5))/ 2×3

x= (-2+-8)/6

now we just seperate the equations so that we have the two roots. and then just solve!

x= (-2-8)/6 -> x= -5/3

x= (-2+8)/6 -> x=1

hope this helps! best wishes and best of luck!!

What is the volume?
9 ft
4 ft
2 ft
HELPPPP

Answers

Answer:

72?

Step-by-step explanation:

V=whl=4 x 2 x9=72

Answer:

72 cubic feet

Step-by-step explanation:

*this formula doesn’t work for all shapes*

Volume = length x width x height

Volume = 4 x 2 x 9

Volume = 72 cubic feet

A game involves correctly choosing the 5 correct numbers from 1 through 18 that are randomly drawn. What is the probability that a person wins the game, if they enter a) once? b) 7 times with a different choice each time?

Answers

Answer:

[tex]=\frac{1}{8568}\ = .00011\\\ =\frac{7}{8568} = .00081[/tex]

Step-by-step explanation:

[tex]5/18\cdot \:4/17\cdot \:3/16\cdot \:2/15\cdot \:1/14=\frac{1}{8568}[/tex]

Help anyone can help me do the question,I will mark brainlest.​

Answers

Answer:

a) 30

b)600pi

Step-by-step explanation:

For the first questions, since the arc is 240°, the area of the sector and circumference will be 240/360 or 2/3 of the total of the circles'. Therefore 125.6 x 3/2 is the circumference, which is 188.4. When we divide this by 6.28, we get 30

Now, since the area is pi r^2 where we know that r=30, we get 900pi as the area of the whole thing, however since the sector is 2/3 of the whole circle, 2/3 x 900pi = 600pi

A person walks away from a pulley pulling a rope slung over it. The rope is being held at a height 10 feet below the pulley. Suppose that the weight at the opposite end of the rope is rising at 4 feet per second. At what rate is the person walking when s/he is 20 feet from being directly under the pulley

Answers

The image of this question is missing and so i have attached it.

Answer:

dd/dt = 4.47 ft/s

Step-by-step explanation:

From the image attached, let's denote the following;

d = horizontal distance beneath pulley

h = height of pulley

l = diagonal from the pulley to the head of the person

v = velocity of rope rising

Using pythagoras theorem;

l² = d² + h²

Differentiating with respect to time and considering h = c^(te) gives;

2l(dl/dt) = 2d(dd/dt)

We are given;

d = 20 ft

h = 10 ft

v = 4 ft/s

We know that velocity in this case is change in diagonal distance with time. Thus;

v = dl/dt = 4 ft/s

From earlier, we saw that;

2l(dl/dt) = 2d(dd/dt)

Thus, reducing it gives

(dl/dt)(l/d) = dd/dt

Now, l² = d² + h²

l = √(d² + h²)

Also, v = dl/dt = 4

Thus;

4(√(d² + h²))/d = dd/dt

4(√(20² + 10²))/20 = dd/dt

dd/dt = 4.47 ft/s

20) solve:
[tex] {8}^{2} + 2 = [/tex]
21) solve:
[tex]4(2x + 5y = [/tex]
22) simplify the expression
[tex]4( {2}^{2} + 30) - 4 = [/tex]

Answers

Ques20. 66
Ques21. 8x+20y
Ques22. 132

Match the answers……………..

Answers

9 in 8956 = 900

9 in 95675 = 90000

9 = 9 in 124569

9 in 68795 = 90

90000 = 9 in 2549652.........

hope it helps...

You order CDs for $14.25 each and the website charges $4.50 for each shipment.
The expression $14.25p + $4.50 represents the cost of p CDs. Find the total cost for
ordering 4 CDs.

Answers

Answer:

$61.50

Step-by-step explanation:

14.25(4) + 4.50

= 57.00 + 4.50

= 61.50

A box contains a yellow ball, an orange ball, a green ball, and a blue ball. Billy randomly selects 4 balls from the box (with replacement). What is the expected value for the number of distinct colored balls Billy will select?

Answers

Answer:

[tex]Expected = 0.09375[/tex]

Step-by-step explanation:

Given

[tex]Balls = 4[/tex]

[tex]n = 4[/tex] --- selection

Required

The expected distinct colored balls

The probability of selecting one of the 4 balls is:

[tex]P = \frac{1}{4}[/tex]

The probability of selecting different balls in each selection is:

[tex]Pr = (\frac{1}{4})^n[/tex]

Substitute 4 for n

[tex]Pr = (\frac{1}{4})^4[/tex]

[tex]Pr = \frac{1}{256}[/tex]

The number of arrangement of the 4 balls is:

[tex]Arrangement = 4![/tex]

So, we have:

[tex]Arrangement = 4*3*2*1[/tex]

[tex]Arrangement = 24[/tex]

The expected number of distinct color is:

[tex]Expected = Arrangement * Pr[/tex]

[tex]Expected = 24 * \frac{1}{256}[/tex]

[tex]Expected = \frac{3}{32}[/tex]

[tex]Expected = 0.09375[/tex]

Find a 2-digit number smaller than 50, the sum of whose digits does not change after being multiplied by a number greater than 1

Answers

The only 2-digit number that is lesser than 50 and the sum of its digits remain unaffected despite being multiplied by a number < 1 would be '18.'

To prove, we will look at some situations:

If we add up the two digits of 18. We get,

[tex]1 + 8 = 9[/tex]

And we multiply 18 by 2 which is greater than 1. We get,

[tex]18[/tex] × [tex]2 = 36[/tex]

The sum remains the same i.e. [tex]3 + 6 = 9[/tex]

Similarly,

If 18 is multiplied to 3(greater than 1), the sum of the two digits comprising the number still remains the same;

[tex]18[/tex] × [tex]3 = 54[/tex]

where (5 + 4 = 9)

Once more,

Even if 18 is multiplied to 4 or 5(greater than 1), the sum of its digits will still be 9.

 [tex]18[/tex] × [tex]4 = 72[/tex]

[tex](7 + 2 = 9)[/tex]

[tex]18[/tex] × [tex]5 = 90[/tex]

[tex](9 + 0 = 9)[/tex]

Thus, 18 is the answer.

Learn more about 'numbers' here: brainly.com/question/1624562

In the PQRS triangle PQ=QR, QR side extended to S Show that PQ+RS=QS. -S Q R

pls explain too

Answers

Answer:

Step-by-step explanation:

from the picture:

QP = QR

and

QR = RS

so

PQ + RS = QS

Other Questions
An educational software company wants to compare the effectiveness of teaching about supply and demand curves between computer animation presentations and textbook presentation. The company tests the economic knowledge of a number of first-year college students, then randomly divides them into two groups. One group uses the animation and the other studies the text. The company retests all the students and compares the increase in economic understanding between the two groups. Is the study described above an experiment? Why or why not? A publisher reports that 54% of their readers own a personal computer. A marketing executive wants to test the claim that the percentage is actually different from the reported percentage. A random sample of 200 found that 44% of the readers owned a personal computer. Is there sufficient evidence at the 0.10 level to support the executive's claim cho dy s a[] c n phn t v dy s b[]c m phn t l cc cc s nguyn dng nh hn 1000 . Gi tp hp A l tp cc s khc nhau trong a[], tp hp B l cc s khc nhau trong b[] What is the last country to introduce television? The points A,B,C and D divide the line segment AD in the ratio 4:3:1 , respectively , and AD = 72cm . What is the length of BD? Milltown Company specializes in selling used cars. During the month, the dealership sold 32 cars at an average price of $16,000 each. The budget for the month was to sell 30 cars at an average price of $17,000. Compute the dealership's sales price variance for the month. President Kennedy and Soviet premier Nikita Khrushchev: Group of answer choices engaged in a dangerous game of tactics and bluffs because the US wanted the Soviets to remove missile sites from Cuba decided which non-aligned nations would ally with the US and which would ally with the Soviets together planned and carried out the Bay of Pigs invasion became good friends after their Vienna summit meeting The blueprints of a house have a scale factor of 30. If one side of the house measures 4 inches on the blueprint, how long is the actual side length (in feet)?A. 7.5 feetB.10 feetC. 90 feetD. 120 feet A recipe for eight flapjacks needs 2oz of butter, 3oz of sugar, and 4 oz of rolled oats. How many flapjacks can I make if I have 14 oz of butter, 15 oz of sugar, and 16 oz of rolled oats? The curve y=2x^3+ax^2+bx-30 has a stationary point when x=3. The curve passes through the point (4,2). (A) Find the value of a and the value of b.#secondderivative #stationarypoints Simplify 2m^2 2m + 3m^2 Jobs contends that you need to love what you do in order to be great at it. Do you agree or disagree? Why Which school faculty member should you talk to about enrolling in an APcourse? Im having a lot of trouble, can someone guide me, step by step? Porque crees que las personas llegan a consumir sustancias psicoactivas About lasiks eye surgery, Provide an example of changes over the years that may have prompted the public to opt for this procedure to correct their vision. Andrea created a picture of an aquarium, which she plans to give to her teacher to hang in the classroom. Andrea was very excited by the idea that her teacher would hang up her picture on the wall, especially since the reason Andrea had made it was to show her teacher she could draw all sorts of fish from memory. In which of Erikson's stages do children realize that they can obtain the recognition of teachers and parents by producing things Have you any idea............ ? a. What is his name b. what his name is My friend prefers coffee C. what name tea PLS HELP 40 pointsIf you had the opportunity to fight against machismo, what would be some of the actions you would take to combat this social phenomenon? The measures of two angles of a triangle are 36 degree and 75 degree . The length of the shortest side of a triangle is 10 cm . The length of longest side of the triangle is:?